You are on page 1of 30

1.

Sol.

If high tides are observed on a particular date at a place in India, one may observe
(a) low tides in the United States
(b) low tides in Australia
(c) high tides in both United States and Australia
(d) low tides in India after half a month.
(d)
High tides & low tides occur once in fortnight.

2 sin 2 cos 2 sin2 cos2

2.

What is the value of x if x =

Sol.

(a) 4 sin
(b) 2 sin 2
(c)
x (2sin2 cos 2)(sin2) (cos 4)( cos 2)
=

cos 4

2 sin cos
(c) cos2

(d) cos6

(c) (ab + bc + ca)

(d) (a2 + b2 + c2)

sin 4 sin2 cos 2 cos 4

= cos2

3.

Sol.

xa

xb

xc

If

(a) (a + b + c)
(b)
xa
b

0 then x is given by

(b) (a + b + c)
c

xb

xc

C1 C1 C2 C3

(x a b c) 1 x b
1

xc

R1 R1 R2 & R2 R2 R3

0 x
(x a b c) 0

0
x 0
xc

x2

(x + a + b + c) = 0
x (a b c)

4.

Sol.

A particle of mass 1 kg is moving along a line y = x +2 (here x and y are in metre) with speed
2 ms-1. The magnitude of angular momentum of particle about the origin is
(a) 4kg m2s1
(b) 2 2 kgm2 s 1
(c) 4 2 kg m2 s1
(d) 2kg m2s1
(b)

www.examrace.com

Angular momentum about origin = 1 (2) cos x 2


= 4 cos
= 4

5.

2
2 2

2 2 kg m 2 s 1

If R is the radius of the earth and g is the acceleration due to gravity, the mean density of earth is
3g
4RG
3R
4G
(a)
(b)
(c)
(d)
4RG
4gG
3gR
3gR
(d)

Sol.
As g

6.

Sol.

7.
Sol.

GM
R2

gR 2
3g

4
4GR
G R 3
3

Compared with others, the planets Earth and Venus have nearly same size and density. But
strength of the magnetic field of Venus is negligibly small compared to that of the Earth. This is
due to
(a) larger orbital speed of Venus
(b) the absence of atmosphere around Venus
(c) absence of metallic rocks inside Venus
(d) very slow rotation of Venus
(c)
Some physical attributes of venus is very similar to earth but absence of metallic rocks inside
makes its magnetic field weaker compared to earth
The sum of the cubes of three successive integers is always divisible by
(a) 4
(b) 8
(c) 9
(d) 12
(c)
Tn = n3 + (n + 1)3 + (n + 2)3
Tn + 1 = (n + 1)3 + (n + 2)3 + (n + 3)3
So Tn + 1 Tn
= (n + 3)3 n3
(n + 3 n) ((n + 3)2 + n2 + n(n + 3))
3(n2 + 9 + 6n + n2 + n2 + 3n)
3(3n2 + 9n + 9)

www.examrace.com

8.

Sol.

9(n2 + 3n + 3)
Divided by 9
The circles x2 + y2 = 400 and x2 + y2 10x 24y + 80 = 0 are in same plane. Which of the
following statement is true?
(a) They do not touch each other
(b) They intersect each other in two points.
(c) They touch each other externally
(d) They touch each other internally
(b)
c1 (0, 0)
r1 = 20
c2 (5, 12)
c1c2 = 13

r2 = 52 122 80 89
r1 + r2 > 13

r1 r2 20 89
r1 + r2 > c1 c2 > r1 r2

They intersect each other in two points.


9.

Sol.

A 40 kg slab S rests on frictionless floor and a 10 kg block B rests on the top of the slab. The
coefficient of friction between them is s 0.45 and k 0.4 . If the force applied on the block is
75N, the accelerations of the slab and the block respectively are (g = 10 ms -2).

(a) 3.5 ms-2, 1.0 ms-2


(c) 3.5 ms-2, 3.5 ms-2
(d)

(b) 1.0 ms-2, 1.0 ms-2


(d) 1.0 ms-2, 3.5 ms-2

ms = 0.45
mk = 0.4

75N

S
Friction require to move together = 40

75
= 60N.
50

But fL (available) = 0.4 x 10g = 45 N


Relative motion takes place between slab and block
aslab =

aBlock =

10.

40
1m / s 2
40

75 40
3.5 m / s 2
10

Select the wrong one (here N is newton, A is ampere, J is joule, m is meter, s is second and kg is
kilogram).
stress
Nm2
(a)
(b) Surface tension = Jm -2
strain
www.examrace.com
(c) Capacitance = A2s4Kg-1m-1
(d) Force = K gms-2

Sol.

(c)
(a)

stress
N
2 (Correct)
strain m

(b) Surface tension =

surface energy
J
2 (Correct)
area
m

(c)

Q
Q
Q2
A 2T 2

V W / Q W ML2 T 2

A 2s 4 kg 1m 2
A 2 s 4 kg 1m 1 is wrong
(d) Force = ma = kg m.s2(Correct)
11.

If cos20 - sin20 = k then cos40 is equal to


(a)

Sol.

2k k 2

(b) k (2 k)2

(c) k 2 k 2

(d) k 2 2k k 2

(c)
k = cos 200 sin 200
k2 = cos2 200 + sin2 200 2 sin 200 cos 200
k2 = 1 sin 400
sin 400 = 1 k2

cos400 1 (1 k 4 2k 2 )
=

2k 2 k 4

= k 2 k2
12.

Sol.

Sun spot are used in the study of


(a) rotation of the sun
(b) size of the sun
(c) variation in the luminosity of Sun
(d) variation in the gravitational field around the Sun.
(c)
Sun spots are temporary phenomena on the sun that appear visibly as dark spots compared to
surrounding regions

13.

Which of the following statements are correct


2 tan
(i) sin2
1 tan 2

(iii)
Sol.

sin 2 cos2 sin 2

(a) (i) & (iv)


(c)
2 tan
sin2
sec 2
2 sin cos
(i) is true
(ii) is true

(b) (i), (ii) & (iii)

(ii) sin 2 2 tan cos 2

tan 2
sin 2
sin 2
(c) (i), (ii) & (iv)
(iv)

(d) All

www.examrace.com

False

(iii)

cos 2 sin2
sin2
(iv)
sin2
cos 2 sec 2

14.
Sol.

15.

Sol.

is true

Five (5) parallel lines in a plane are intersected by set of four (4) parallel lines. The number of
parallelogram is
(a) 12
(b) 20
(c) 25
(d) 60
(d)
Number of parallelogram is 5 C2 .4 C2
= 10 6 = 60
A horizontal platform at rest starts ascending from the ground with a constant acceleration of
1 ms-2. After 2 seconds a stone is thrown vertically upwards from the platform with a speed of
21ms-1 relative to the platform. The maximum height from the ground attained by the stone is
(g = 10 ms-2)
(a) 2.8 m
(b) 2.2 m
(c) 2.4 m
(d) 2.6 m
(a)
at t = 2 sec plat-form is at

1
1 4 2m
2

initial velocity of particle (u) = 2 + 2 = 4 m/s


From 2 m maximum height is

42
16

0.8
2 10 20

From ground ----- 2.8 m

16.

The efficiency of carnots heat engine is 0.5 when the temperature of the source is T 1 and that of
sink is T2. The efficiency of another carnots heat engine is also 0.5. The possible temperature of
the source and sink of the second engine are respectively.
T
(a) T1 + 5, T2 5
(b) T1 + 10, T2 10
(c) 2T1, 2T2
(d) 2T1, 1
2
(c)

Sol.

T
1 2
T1
to have constant

17.

Sol.

T2
ratio should be constant i.e., (2T 1, 2T2)
T1

Two hypothetical stars A and B of same size have apparent magnitudes of 4.5 and 1.5 and real
magnitudes of 1.5 and 4.5 respectively. We conclude that
(a) A is much hotter and closer to earth than B
(b) B is much hotter and closer to earth than A
(c) A is much hotter than B and B is much closer to earth than A
(d) B is much hotter than A and A is much closer to earth than B
(d)
Lower the apparent magnitude higher is the brightness of star from earth hence A is nearer to
earth while real magnitude of B is less hence it is hotter compared to A
www.examrace.com

18.

Sol.

Find the value of 1 8 27


1 32 243
(a) 36
(b) 216
(d)
R2 R2 R1 & R3 R3 R1
1

24

(c) 1296

(d) 61

0 30 240

1 2 3
6 30 0 1 4
0 1 8
180 1(4) 720 6!

19.

Find the value of tan1 2 + tan1 3

(a)
(b)
4
4
(c)

(c)

3
4

(d)

5
6

Sol.

tan 1 2 tan 1 3
2 3
tan 1
1 6
tan 1 (1)
3

4
4
a 4b2

20.

A physical quantity y

Sol.

and d are 2 %, 3%, 4% and 5% respectively. The error in y will be


(a) 6 %
(b) 11 %
(c) 12 %
(d)

cd
4

1/3

has four variables a, b, c and d. The percentage error in a, b, c


(d) 22 %

y
b 1 c 4 d
a
% 4
%
2

y
b 3 c 3 d
a

1
4
4(2) 2(3) (4) (5)
3
3
=8+6+8
= 22 %

21.
Sol.

Two identical drops of water fall through air with a terminal velocity 2 ms 1. If the drops merge to
form a single drop, its terminal velocity is
(a) 3.17 ms1
(b) 1.58 ms1
(c) 2.52 ms1
(d) 4 ms1
(a)

vr2

v1 r1

v 2 r2

www.examrace.com

r 1
4
4
2 r13 r23 1
3
3
r

2
2

v1 1 3

v2 2
v2 v1

22.
Sol.
23.
Sol.

2
(2) 3

3.17 m / s

A part from the earth, Aurora phenomena is observed on which of the following planet
(a) Venus
(b) Mars
(c) Mercury
(d) Jupiter
(d)
Apart from earth Auroras have been observed on. Jupiter and Saturn
The mean of the five numbers is zero. Their variance is 2. If three of these numbers are 1, 1 and
2. The remaining numbers are
(a) 5 & 3
(b) 4 & 2
(c) 3 & 1
(d) 2 & 0
(d)
Let numbers be x & y
1 1 2 x y
0
5
x y 2

xi

(Mean)2
n
(1)2 (1)2 (2)2 x 2 y 2
2
0
5
10 = 6 + x2 + y
(x + y)2 = 4
x2 + y2 = 4
x + y = 2 Hence x = 2 & y = 0
Variance =

24.
Sol.

25.
Sol.

Find the digit at the unit place of 272015 + 212015 232015


(a) 7
(b) 3
(c) 5
(a)
(27)4 503 3 (21)4503 3 (23)4503 3
The digit at unit place will be 3, 1, 7 respectively
So, the unit place is 10 + 3 + 1 7 = 7

(d) 1

A cylindrical tube open at both ends has a fundamental frequency of 390 Hz tube is immersed,
vertically in water, the fundamental frequency of air column is
(a) 130 Hz
(b) 390 Hz
(c) 520 Hz
(d) 260 Hz
(d)

l
2

390 v

2l x 390 = v

www.examrace.com

3l '

4 4

' f v

(3l) f = 2l x 390

f = 260 Hz

26.

Three identical rods are joined as shown in the figure. The temperature at the junction is

Sol.

(a) 48c
(d)

(b) 44c

(c) 40c

(d) 36c.

60 T 60 T 12 T

0
R
R
12
60 T + 60 T + 12 T = 0
108 = 3T
T = 360c

27.

Sol.

Sun is at a mean distance of about 27,000 light years from the centre of the milky way galaxy and
completes one revolution about the galactic centre in about 225 million years. The linear speed of
Sun is
(a) 160 km s1
(b) 230 km s1
(c) 30 km s1
(d) 80 km s1
(b)
1 light year = 9.4607 1015m
Speed =

dis tan ce
time

assuming path to be circular

www.examrace.com

v=

2
27000 9.4607 1015
2
R
=
t
225 106 365 24 3600

= 230 km / sec

28.

Find the value of tan1

Sol.

x
4
(c or d or cd)
cos x
tan1
1 sin x
(a)

cos x
1 sin x

(b)
2

(c)

4 2

(d)

4 2

x
sin

2 2
tan1

1 cos x
2

x
x
2sin cos

4 2
4 2
tan1

x
2

2cos

4 2

x
tan1 tan1
4 2

= 4 2

29.
Sol.

30.

Sol.

If log 30 = 1.4771, log 40 = 1.6021, log 50 = 1.6990, log 45 = x then find x.


(a) 1.6542
(b) 1.6232
(c) 1.6532
(d) 1.6832
(c)
log(5 9)
log 5 + 2 log 3
0.6990 + 0.4771 2
0.6990 + 0.9542
= 1.6532
The figure shows a pressure vs temperature (P T) graph for a given mass of an ideal gas
undergoing a process from A to B. In this process, volume of the gas

(a) is increasing
(a)

(b) remains constant (c) is decreasing

(d) cannot be predicted.

P = MT + C
Pv = nRT

nRT
MT C
v
www.examrace.com

T
v nR

MT C
(MT C)(1) T(M)
dv
nR

dT
(MT C) 2

f(x, t) = f(x-vt, 0)

dv
C
nR
ve
2
dT
(MT C)
31.

Sol.

A window is 50 cm high. A stone starts falling from a height of 40 cm above the window. It crosses
the window in
3
5
4
1
(a) S
(b) S
(c) S
(d) S
7
7
7
7
(a)

v2 = v1 + at where AB = height of window

32.
Sol.

v1 =

2 9.8 0.4

v2 =

2 9.8 0.9

t=

v2 v1
=
a

2 9.8 0.9 2 9.8 0.4


1
= sec.
9.8
7

Light from the nearest star proxima centauri takes 4.24 light years to reach earth. The stellar
parallax of this star is about
(a) 1.30 sec
(b) 0.77 sec
(c) 13.8 sec
(d) 0.24 sec
(c)
1 parsec (pc) = 3.0857 1016 m = 3.261 light year
1 light year = 9.4607 1015 m

www.examrace.com

Stellar parallax =

1
1.3009 pc or 4.243light year

= 0.7687
33.

Sol.

x2
y2

1 represents
25 a 49 a
(a) an ellipse if a > 25
(b) a hyperbola if 25 < a < 49
(c) a hyperbola if a > 49
(d) an ellipse if 25 < a < 49
(b)
The equation

x2
y2

1
(25 a) 49 a
25 a < 0,
a > 25,
25 < a < 49

34.

Sol.

4a>0
49 > a

sin cos
x

The value of the determinant sin x


1 is
cos
1
x
(a) Independent of
(b) Independent of x
(c) Independent of and x
(d) none of these is true
(a)

x( x 2 1) sin( xsin cos ) cos ( sin x cos )


x 3 x x(sin 2 cos 2 )
x3
35.

Two similarly charged spherical conductors are suspended by non conducting threads of length l
from a horizontal support. If this is taken to a place of zero gravitational effect, then angle between
the threads and the separation between them are
(a) 0, 2l

Sol.

(b) 180, 2 l

(c) 120, l /2

(d) 60,

3 / 2 l.

(b)
In absence of gravity , only electric repulsive effect takes place acts

36.

Sol.

A cylindrical object of volume 0.4 m3 floats in water with 20 % of its height seen above water. The
minimum force required to be applied on the object so that it just gets immersed in water is
(Density of water = 103 kg ,m 3 g = 10 ms2)
(a) 32000 N
(b) 800 N
(c) 2400 N
(d) 4800 N
(b)

www.examrace.com

v = 0.4m3

0.8h

V = 0.4 = Ah (Given)
F = Extra Buoyancy force =

VExtra submerged g

= 103 x A x 0.2h x 10
= 103 x 0.4 x 0.2 x 10
= 800 N.

37.

Sol.

38.
Sol.

39.

Sol.

The International Space Station (ISS) launched in 1998 its satellite orbiting earth with perigee at
422 km and apogee at 425 km above mean sea level. Given mean radius of the earth is 6371 km
and acceleration due to gravity is 9.8 ms2, the distance covered by ISS in one hour is about
(a) 27500 km
(b) 766 km
(c) 4150 km
(d) 2660 km
(a)
Mean height is around423 km
6371 + 423
6800
Distance from centre of earth
We know it takes around 1.5 hrs for near earth satellite
2
1 hr implies
of perimeter
3
2
2
2R 2 6800km

3
3
= 28000 km
Hence (a)
Find the 99th term of the sequence 7, 23, 47, 79, 119, .
(a) 35999
(b) 379999
(c) 37799
(d)
7, 23, 47, 79, 119,
By method of difference
Tr = 4r2 + 4r 1
So, T99 = 4 992 + 4 99 1 = 39599

(d) 39599

forty two cube with 1cm edge each are glued together to form a solid rectangular block. If the
perimeter of the base is 20 cm. Find the height of that rectangular block.
(a) 2 cm
(b) 3 cm
(c) 6 cm
(d) 7 cm
(a)
Volume of the rectangle block = 13 x 42 cm3
= 42 cm3

www.examrace.com

2 (l + b) = 20
l + b =10
All sides will be integers
l b h = 42
3 x 7 h = 42
h=2
40.

Sol.

41.

Sol.

Out of the following expressions


a
2
(ii) y a x t
(iii) y a x t
(i) y
x t
The expression/s that represent/s progressive waves is/are
(a) (i) and (ii) only
(b) (ii) and (iii) only
(c) (i), (ii) and (iii) only
(c)
f(x, t) = f(x vt, 0)

(iv) y a x2 t 2

(d) all

There are two open organ pipes of exactly the same length and material but of different radii.
Then,
(a) narrow pipe has lower frequency
(b) both the pipes have exactly the same frequency
(c) wider pipe has lower frequency
(d) either has lower frequency depending on the amplitude of sound waves.
(c)
end correction depends on radius
large radius
larger end correction
larger wavelength
smaller frequency
(c) is correct.
Read the following passage and answer the questions number 42 to 44
A star of mass greater than about 5 times the mass of the Sun (M 0) dies as black hole. When the
nuclear burning stops, the star starts collapsing under its own gravity. On attaining a certain radius,
its escape velocity becomes equal to the speed of light and it is called the Schwarzschild radius
Rs. An event horizon is said to be formed when No information can be obtained by
electromagnetic waves from within. For a non-rotating black hole, the Schwarzschild radius itself
. 2 / kg2 , c 3 108 ms 1 , M0 21.6 1030 kg and
forms the event horizon. ( G 6.674 10 11 Nm
Mearth 5.972 1024 kg )

42.
Sol.

The Schwarzschild radius of a star of mass 8M0 is about


(a) 3 km1
(b) 256 km
(c) 140 km
(b)

da 2L

dd 2' m

(d) 480 km s-1

www.examrace.com

C=

2Gm
escape speed
R
2 6.674 10 11 8 21.6 10 30
R

3 108 =

9 1016 =

1.6 6.674 1019 21.6


R

R = 256 km
43.

Sol.

44.
Sol.

After a massive star attains the Schwarzschild radius R s, on further gravitational contraction, the
radius of event horizon of a non rotating black hole
(a) decreases
(b) increases
(c) remains constant
(d) becomes infinite
(c)
Rs is the radius at which escape velocity become C hence Rs is constant for a given star.
Suppose earth starts shrinking to become a black hole then the event horizon will
(a) 8.9 km
(b) 2.35 m
(c) 0.89 cm
(d) 12.45 cm
(c)

24 m
R

x=

2 6.674 10 11 5.972 10 24
R

(3 108)2 =

R=

8.857 1013
1016

R = 0.89 cm
45.
Sol.

46.
Sol.

51

32

1532 is a multiple of

(a) 765
(b) 2178
(c) 2826
(c)
5132 1532
(5116 1516) (5116 + 1516)
(518 158) (518 + 158) (5116 + 1516)
(514 154) (514 + 154) (518 + 158) (5116 + 1516)
(512 152) (512 + 152) (514 +154) (518 + 158) (5116 + 1516)
(36) (66) (512 + 152) (514 + 154) (518 + 158) (5116 + 1516)
Hence divisible by 2826
100 6021 4; 100 4771 3 and 10 x 72 find x.
(a) 1.85735
(b) 1.75725
(c) 1.87525
(a)
100.6021 = 4
100.4771 = 3
log
4
10
=4
10log 3 = 3
log 4 = 0.6021
x
10 = 72
log 3 = 0.4771
10x = 72
x = log 72
= 2 log 3 + 3 log 2
0.6021
= 2 0.4771 + 3
2

(d) 2956

(d) 1.87255

www.examrace.com

= 0.9542 + 0.90315
= 1.85735
47.
Sol.

48.

Sol.

49.

Find the smallest natural number n such that 65n2 1 is a perfect square
(a) 13
(b) 16
(c) 17
(b)
65n2 + 1 = k2
k 2 1 (65 1)2 1 652 2 2 65
= 652 2

n2
65
65
65
So, minimum value of = 2
Hence n = 16

(d) 19

Two litres of milk purchased from a vendor, has a density of 1018 kgm -3. If the density of pure milk
from that source is 1030 kgm-3 and that of water used is 998 kg m -3, the amount of water mixed in
he sample is
(a) 0.750 litre
(b) 0.667 litre
(c) 0.480 litre
(d) 0.375 litre
(a)
water milk = 2L = V, m = 1018 kg/m3
Density of milk = 1 = 1030, volume of milk = V2
Density of water = 2 = 998, volume of water = V2
mass of water + mass of milk = mass of (milk + water)
2 V2 + 1 V1 = Vfm
(i)
998 V2 + 1030 V1 = 2036
V1 + V2 = 2
(ii)
998 V2 + 1030 [2 V2] = 2036
- 32 V2 + 2060 = 2036
V2 = 0.75 L
Particle A starts from origin with a constant acceleration 3ms -2. Its initial speed is zero. Particle B
starts from x = 48 m and moves with constant speed of 6ms -1. If both the particles move along x
axis and start at the same instant, then identify the correct graph which depicts motion of both the
particles.

(a)

(b)

(c)

(d)

(a)
Sol.

aA 3ms2 , uA 0

www.examrace.com

6ms (Constant)
B
1
S A uA t a A t 2
2
1
x A 0 (3)t 2
2
xA = 1. 5t2

x = 48, u

50.

SB uB t

xB 48 = 6t
xB = 6t + 48

In the situation shown in adjacent figure, coefficient of friction between block B and wedge A is 0.5.
Wedge A is moved horizontally with uniform acceleration a. Then identify the correct statement
(take tan 370 3 4 ).

(a) Friction on the block is zero if a = g


(b) Friction on the block is upward along the incline if a > g
(c) Friction on the block is downward along the incline if a
Sol.

51.
Sol.

(d) Friction on the block is kinetic in nature if a 2 g


(d) If = g tan , tnen f = 0
3g
for f = 0
a
4
3g
then f is down the incline
If a
4
3g
a
then f is up the incline
4
(d) is correct.
Find he sum of all three-digit perfect square numbers
(a) 9131
(b*) 10131
(c) 10231
(b)

g
2

(d) 11031

The sum of 102 112 122 132 142 ......... 312

1 2
2

.......... 312 12 22 .......... 92

31 32 63 9 10 19

6
6
10416 285

10131
52.

Which of the following statements are true


8
84
cos 1
sin 1 0.6
(i) sin 1
17
85

(ii) sin 1

8
84
cos 1
tan 1 0.75
17
85

www.examrace.com

8
84
cos 1
cot 1 0.75
17
85
(a) (i) & (iv)
(b) (i), (ii) & (iii)
(c)
8
84
sin 1
cos 1
sin 1 (say)
17
85

(iii) sin 1
Sol.

8
84
cos 1
cos 1 0.80
17
85
(c*) (i), (ii) & (iv)
(d) All
(iv) sin 1

8
84

Taken sin both side sin sin 1


cos1 sin sin 1

17
85

8 84 15 13

17 85 17 85
672 195

17 85
867

17 85
0.6
Hence, ans is sin-1 (0.6), cos-1 (0.8) & tan-1 (0.75)
(i), (ii) & (iv) are true.

Read the following passage and answer the questions number 53 to 56


The Mars orbital Mission (MOM) space craft launched on 9th November 2013 reached the
expected Martian orbit on 24th September 2014. Now it is revolving the planet Mars in a highly
elliptic orbit. At the closest position the space craft is at a height of 421.7 km above the Martian
surface, its farthest distance from the planets surface being 76993.6 km. The mean radius of Mars
is about 3390 km. One of the two natural satellites of Mars is Phobos revolves the planet in a
nearly circular orbit with a mean radius of 9377 km once every 7.66 hours (mass of mars = 6.39 x
1023 kg and G = 6.647 x 1011 N.m2/kg2).
53.
Sol.

54.
Sol.

The semi major axis of MOM's orbit is about


(a) 42100 km
(b) 38500 km
(a)
2a = 421.7 + 2 x radius of mars + 76993.6
a = 42096.65 km
The semi minor axis is about
(a) 21008 km
(b) 17510 km
(b)

(c) 40000 km

(d) 36800 km

(c) 4207 km

(d) 19503 km

ae a 421.7 3390

e = 0.909

b2 a 2 (1 e2 )
b a (1 e 2 )

b = 17545.7 km
55.
Sol.

The period of revolution of the space craft in hours is about


(a) 36.4
(b) 26.5
(c) 72.8
(c)
2

T1
r1
T r
2
2

(d) 53.0

From Keplers law

www.examrace.com

7.66
9377
T r
2
2
58.67

0.0110

T22

T22

56.
Sol.

58.69
73.031
0.011

The maximum speed of the space craft in its orbit is about


(a) 2.15 kms1
(b) 1.85 kms1
(c) 7.89 kms1
(Bonus)

(d) 1 kms1

Maximum speed would be at A


Vmax

GM 1 e

a 1 e

e = 0.909

b2
e 1 2 Where b & a are semi minor and semi major axes.

a = 42100 km.
By putting all values
Vmax 4.31 km / s.

Ans : No option is correct hence BONUS.

57.

If y =

cos(x)
dy
then
is given by
sec(x)
dx

(a) sin (2x)


Sol.

(b) sin (2x)

(c) sin(x) cosec(x)

(d)

sin(x)
cos ec(x)

(b)

cos x
1
cos x

y cos 2 x

dy
2 cos x sin x
dx
sin 2x

www.examrace.com

58.

Sol.

The owner of a milk store finds that he can sell 980 litres of milk each week at Rs 42/litre and 1220
litres of milk each week at Rs 48/litre. Assuming the linear relationship between the selling price
and the demand, how many litres could he sell weekly at Rs 51/litre.
(a) 1340
(b) 1430
(c) 1450
(d) 1540
(a)
Since we have to assume linear relationship
Z Ax B
Here A & B are constants.
Now, 980 = A (42) + B
(1)
1220 = A (48) + B
(2)
Solving (1) & (2)
B 700

A 40

Now finally Z 40 51 700


= 1340
59.
Sol.

Two hypothetical main sequence stars A and B have their radius in the ratio 1:2 and their surface
temperature in the ratio 2:1. Then the ratio of their luminosities respectively are in the ratio
(a) 4:1
(b) 1:1
(c) 2:1
(d) 1:16
(a)
Luminosity L AT 4

L1 4r 2 2T

Ratio

L 2 4 2r

L1 r 2 16 T 4 4

L2
1
4r 2 T 4

Ratio is 4 : 1
60.

Sol.

Find the value of if cos3 cos cos 2

(a)
(b)
2
(d)
cos3 cos cos 2
2cos 2 cos cos 2 0

(c)

2
2

(d)

cos 2 (2cos 1) 0
cos 2 0

2
2

61.

cos

1
2

Consider an isolated system of interacting particles. Then indentify the correct statement
(i) Total mechanical energy of the system must be conserved.
www.examrace.com
(ii) Total linear momentum of the system must be conserved.

Sol.

62.

Sol.

(a) (i) is true and (ii) is false


(b) (i) is false and (ii) is true
(c) (i) and (ii) both are true
(d) (i) and (ii) both are false
(b)
For isolated system :
(i) Mechanical energy may not remain conserved. For interacting particles during collision energy
may be lost to heat energy.
(ii) Momentum of system will remain conserved.
A gaint wheel of mass M has a rim on which a person can walk. Consider a case where, in a
person of mass m walks on the rim at a constant speed and the wheel rotates in opposite
direction with constant angular speed. Further the speed of the person with respect to ground
is zero. Then the net force exerted by axle on the wheel.
(i) Is equal to (m + M) g
(ii) ls greater than (m + M) g
(iii) Depends on angular speed of the wheel
(iv) Depends on radius of the wheel
(a) (i) is true
(b) (i) and (ii)are true
(c) (ii) and (iii) are true
(d) (ii), (iii) and (iv) are true
(a)

For Man :

Since w.r.t. ground it is at rest.


N mg
So, no centripetal force or acceleration.
For wheel :

www.examrace.com

N1 N Mg

N1 M m g
Force exerted by axle on wheel = (M + m) g

No effect of angular speed or radius of wheel.


63.

Sol.

A particle is projected with a velocity, 20 ms 1 at angle of 60. Then radius of curvature of its
trajectory at a point where its velocity makes an angle of 37 with the horizontal is
(g = 9.8ms2)
(a) 16 m
(b) 19.53 m
(c) 15.62 m
(d) 25 m
(b)

V cos 370 20 cos 600

4
V 10
5

50
m/s
4

V2
a

www.examrace.com

50 50 1 5

4
4 10 4
6250

= 19.53m
64

64.

A person standing on a plank is pulling a string as shown in the adjacent figure. Floor is rough
and the motion is impending. Tension in the string is denoted by T and coefficient of friction
between plank and floor is . The tension in the string is

mg
1 cos sin
mg
(c) T
1 sin
(a)
(a) T

Sol.

mg
1 cos
mg
(d) T
1
(b) T

Friction

N Tsin mg
T T cos (mg Tsin )
T(1 cos sin ) mg

T
65.

Sol.

mg
1 cos sin

If and are the roots of the equation x2 = (x + 1). How many of the following statements
are true.
(i) 2 =
(ii) = 2
(iii) = 1
(iv) 3 3 = 0
(a) 1
(b) 2
(c) 3
(d) all
(d)

x , 2 (where cube root of unity)


x2 x 1 0
2
Let , ; obvious
66.

Sol.

The numbers a and b are chosen from the set {1, 2, 3, 4, 5, 6, 7, 8, 9, 10} such that a b
with replacement. Find the probability that a divides b
5
49
29
27
(a)
(b)
(c)
(d)
11
100
55
45
(c)
The total number of all possible chosen a & b such that a b is
10 + 9 + . + 1 = 55 &
www.examrace.com
for favorable case
if a = 1 then b = 1, 2, 3..10

if a = 2 then b = 2, 4, 6, 8, 10 similarly
if a = 10 then b = 10
Hence the total number of favorable case is 10 + 5 + 3 + 2 + 2 + 1 + 1 + 1 + 1 + 1 = 27
So the probability is 27/55
67.
Sol.

68.

Sol.

Assuming that straight line works as a plane mirror for a point. Find the image of the point
(1, 2) in the line x 3y + 4 = 0
(a) (2, 1)
(b) (1, 2)
(c) (1.2, 1.4)
(d) (1.2, 1.4)
(c)
The image of point (1, 2) w.r.t. line x 3y + 4 = 0 is
x 1 y 2 2(1 6 4)

1
3
10
x 1 y 2 1

1
3
5
6
x 1.2
5
3 7
y 2 1.4
5 5
There are two pendula as shown in the adjacent figure. Pendulum A is at rest in its
equilibrium position while pendulum B is oscillating. When pendulum B passes through the
equilibrium position, Consider the following two statements

(i) The net force experienced the bob at the equilibrium position is zero in both the cases.
(ii) The acceleration of pendulum B in the tangential direction is zero
(a) Statement (i) and (ii) are correct
(b) Statement (i) is correct while Statement (ii) is wrong
(c) Statement (ii) is correct while Statement (i) is wrong
(d) Statement (i) and (ii) are wrong
(c)

T mg 0

equilibrium

T mg

mV 2
R

Not in equilibrium.

Hence statement (i) is wrong.


Now no net force is acting along the horizontal direction, therefore tangential acceleration is zero in
both cases.
www.examrace.com

69.

An electrical circuit consisting of three identical bulbs and three switches is shown in the adjacent
figure. Let the glowing bulb be presented by one (1) and zero (0) when it is not glowing. The
correct table that represent the circuit operation is

Ans.

(Bonus) No figure is given.

70.

In a polarization experiment a plane polarised light is allowed, to pass through a mica sheet and
then through an analyser. The intensity of light is measured as function of orientation of mica
sheet. The intensity attains minimum at a certain angle and then increases. The data obtained for
three angles close to minima is given by.
Angle

Intensity

-3.6

1.1

0.2

3.6

0.6

Assuming that variation of intensity (1) with respect to angle

is a quadratic function, the angle

at which the intensity of tight is zero is


Sol.

(a) 0.7
(b)

(b) 0.9

(c) 0.3

(d) 0.5

Let I = a 2 +b + c
where is in radian
let 1 = + 3.6o

and 2 = - 3.6o
180
180

let 1 = - 2 =

www.examrace.com

For 3 = 0

I = 0.2

C = 0.2

Now putting values of 1 & 2

and

0.9 = a 2 - b + 0

.(1)

0.4 = a 2 +b

.(2)

Adding 1 and 2
0.13 = 2a 2

0.13(180)2

23.6 3.6

1300
82

Putting value of a in eqn (1)

0.9

0.9

1300
82
1300
8

2 b
(3.6)3.62 b(3.6)

180

83.50
2

Now,

1300
2

83.50
0.2 0
2

13002 334 1.62 0

Now apply the quadratic formulae

b b 2 4ac
2a

334

334 2 4 1300 16 2
2 x1300

Converting into degree

334 103236 9
x
2 x1300

334 321

9
130

13
x 9 0.90
130

www.examrace.com

334 321
2
9 45.360
130
Correct answer is 0.90

71.

Sol.

A cylinder is closed at both ends and has insulating walls. It is divided into two compartments by
an adiabatic (insulating) partition that is perpendicular to the axis of the cylinder. Each
compartment contains 1.00 mol of oxygen that behaves as an ideal gas with ratio of specific heat,
7
y . Initially, the compartments one and two have equal volumes and their temperatures are
5
550K and 250K The partition is then allowed to move slowly until the pressures on its two sides
are equal. The final temperatures in the first and second compartments are respectively
(a) 473.5K & 264.5K
(b) 498.9K & 284.2K
(c) 492.9K & 270.2K
(d) 250K & 550K
(b)

P1Vy = PV1y

...... (i)

P1 V1

P2 V2
nR 550

V
V
1
nR 250 V2
V
1.756

P2Vy = PV2y

..... (ii)

7/5

V1
V2

V1 + V2 = 2V
Using (iii) and (iv)
V1
1.756
V2

....... (iii)
.......... (iv)

2
V
2.756
2 1.756
V1
2.756
then using TVy1 = constant
for Partition number (1)

V2

2 1.756
550 V 2/5 T1

2.756
T1 = 499.17 K

2/5

2V
and for partition number 2 250 V y 1 T2

2.756
T2 = 283 K

72
Sol.

y 1

The reminder that is obtained, if the number (100110011001) 2 is divided by 91 is


(a) 0
(b) 1
(c) 8
(d) 9
(a)

www.examrace.com

Here (100110011001)2
2
100 9 1100110011
91 1100110011

= So it is divisible by 91, Hence remainder = 0


73.
Sol.

If 3x = 7y = 63z then z (2y + x ) is given by


(a) x y
(b*) xy

(c) x2y

(d ) none of these

(b)

3x 7y 63z k

Let
x log 3

74.

Sol.

y log 7 z log 63 log k


log k 2log k log k

log 63 log 7 log 3

z(2y x)

(log k) 2 (2 log 3 log 7)

log 63
log 7 log 3

(log k) 2
xy
log 7 log 3

a and b are unit vectors. is the angle between a and b . Which of the given statement are
true.
2

(i) If a b is unit vector then


(ii) If a b is unit vector then
3
3

(iii) If a b is unit vector then


(iv) If a b is unit vector then
4
3
(a) (i) & (ii)
(b) (iii) & (iv)
(c) (i) & (iii)
(d) none of above
(a)

ab 1

a b 2 a b cos 1

1 1 2cos 1
1
cos
2
2

3
and a b 1

a b 2 a b cos 1

1 1 2cos 1
1
sin
2 2

3
(i), (ii)
75.

When the cube root of 13683*0393208 is divided by 3, the remainder is 2. The digit in place of *
is
www.examrace.com
(a) 5
(b) 6
(c*) 7
(d) 9

Sol.

(c)
Let

(13683 0393208)1/3 3n 2

13683 0393208 (3n 2)

9k 8

13683 0393200 should be divisible by ?


7
76.

Sol.

A glass slab is divided into three lenses l1, l2 and l3 as shown in the adjacent figure. If power of
lens l1 is P1 = 0.25, the power of the lens l2.

(a) 0.5 D
(b) 0.25 D
(c) 0.75 D
(b)
Total power of slab P = 0.
Now, slab divided into three parts so, P1 + P2 + P3 = 0
P1

1
1
1
1
1
1
0.25
f1
R
R1 R2

P3

1
1
1
1 1 2

0.5
1
1

f3
R
0.5R
R1 R2

(d) 0.4 D

Now, 0.25 P2 0.5 0


P2 0.25D

77.

Sol.

An object (O) is placed at a distance of 30 cm from a convex lens of focal length 12 cm and real
image is caught on a screen. A convex mirror is placed between lens and image screen. It's
position, when adjusted forms a final image coincident with O. If the distance between lens and
mirror is 10 cm, the focal length of the convex mirror is
(a) 30 cm
(b) 10 cm
(c) 5 cm
(d) 25 cm
(c)

For convex lens

1
1
1

12 v 30

1 1 1

f v u

www.examrace.com

From above equation v 20cm


Now, distance of image form by convex lens is 10 cm from convex mirror. (because distance
between lens and mirror is 10 cm) To retrace path this point must be centre of curvature of mirror.
fm = R/2 = 10/2 = 5 cm.
78.

Ans.

An electrical heater drawing a current 5 amp is used for melting 5 Kg of ice (at 0). The
temperature of the melt after 1000 sec (latent heat of melting of ice is 80 calg -1, specific heat of
water is 1 calg-1 per degree Celsius and resistance of coil is 40 ).
(a) 10 C
(b) 30 C
(c) 15 C
(d) 0 C
(d)
Heat required to melt ice H = ML = 5000 80 40 10 4 Cal
40 10 4 Cal 168 10 4 J

Now, H i2 R t
This will be equal to heat required to melt the ice, therefore i2 R t 168 10 4 J .
52 40 t 168 10 4 J

t 1680 sec. This time is greater than 1000, so temperature of liquid will be at 0 0.

79.

Sol.

A stone tied to string is whirled in a horizontal plane in a circular path at a rate 5rev s -1. If the
same stone is whirled with same force by a string of half its length, the number of revolution for
one second now is
(a) 80 rev s 1
(b) 10 rev s 1
(c) 50 rev s 1
(d) 40 rev s 1
(c)
F1 F2
mr112 mr2 22

52

22

2 50 rev / s

80.

Consider a small balloon filled with an ideal gas which is submerged in water. Assuming that the
temperature is the same everywhere in the water, the buoyant force on the balloon when it is at a
depth d below surface, in terms of its volume at the surface V0, the atmospheric pressure P0, the
density of water p0, and the acceleration due to gravity g.
p v
p v
dpg p0
p0 v0
(a) FB = 0 0
(b) FB =
(c) FB =
(d) FB = 0 0
p
pg
p0 v0
dpg p0
d
d 0
pg
p0

Sol.

(a)
As temperature of liquid is same everywhere so the process is isothermal, therefore

www.examrace.com

P1V1 P2 V2
P2 P0 , V2 V0 , P1 d g P0

V1

P0 V0
dg P0

Now force of buoyancy at depth


PV
FB V1 g 0 0 g
dg P0

FB

P0 V0
P
d 0
g

www.examrace.com

You might also like